Đến nội dung

sinh vien nội dung

Có 261 mục bởi sinh vien (Tìm giới hạn từ 29-04-2020)



Sắp theo                Sắp xếp  

#557802 Sự kết hợp giữa tích phân và số phức

Đã gửi bởi sinh vien on 04-05-2015 - 08:08 trong Giải tích

Bài toán. Chứng minh rằng số điểm nguyên trong hình lập phương đóng $-n\leq x,y,z\leq n$ thỏa mãn điều kiện $-s\leq x+y+z\leq s$ bằng $\frac{1}{2\pi }\int_{-\pi }^{\pi }\left ( \frac{sin\frac{2n+1}{2}t}{sin\frac{t}{2}} \right )^{3}\frac{sin\frac{2s+1}{2}t}{sin\frac{t}{2}}dt$ trong đó s, n nguyên

Lời giải. Xét hàm sinh của bài toán

     $G(x)=\left ( \frac{1}{x^{n}}+\frac{1}{x^{n-1}}+...+\frac{1}{x}+1+x+...+x^{n-1}+x^{n} \right )^{3}$

  $=...+a_{-k}x^{-k}+a_{-k+1}x^{-k+1}+...+a_{-1}x^{-1}+a_{0}+a_{1}x+...+a_{k-1}x^{k-1}+a_{k}x^{k}+...$

 Dễ thấy khi đó số các điểm nguyên của hình lập phương đóng thỏa mãn điều kiện $x+y+z=m$ trong đó m nguyên và $-3n\leq m\leq 3n$ chính là hệ số $a_{m}$ trong khai triển trên.

 Do đó 

  Số cần tìm = $a_{-s}+a_{-s+1}+...a_{1}+a_{0}+a_{1}+...+a_{s-1}+a_{s}$

 Để tính tổng này ta chú ý đến một kết quả quen thuộc trong giải tích phức:

                         $\int_{-\pi }^{\pi }e^{ikx}dx=2\pi$ nếu k=0 và bằng 0 trong các trường hợp còn lại.

nên nói chung ta thấy:

$\frac{1}{2\pi }\int_{-\pi }^{\pi }G(e^{it})e^{-kxt}dt=a_{k}$, trong đó$-3n\leq k\leq 3n$

 Để thuận tiện trong trình bày ta sẽ đặt $\zeta =e^{it}$ nên ta được :

         $\frac{1}{2\pi }\int_{-\pi }^{\pi }G(\zeta)\zeta ^{-k}dt=a_{k}$

 Ta có

  $\sum_{i=-s}^{s}a_{i}=\frac{1}{2\pi }\int_{-\pi }^{\pi }G(\zeta)(\sum_{i=-s}^{s}\zeta ^{i})dt$

  Không quá khó khăn để chứng minh đẳng thức sau:

                 $\sum_{i=-m}^{m}\zeta ^{i}=\frac{\zeta ^{-\frac{2m+1}{2}}-\zeta ^{-\frac{2m+1}{2}}}{\zeta ^{-\frac{1}{2}}-\zeta ^{\frac{1}{2}}}=\frac{sin\frac{2m+1}{2}t}{sin\frac{t}{2}}$   ( chú ý ta có: $\zeta =e^{it}$

nên $\sum_{i=-s}^{s}a_{i}=\frac{1}{2\pi }\int_{-\pi }^{\pi }\left ( \frac{sin\frac{2n+1}{2}t}{sin\frac{t}{2}} \right )^{3}\frac{sin\frac{2s+1}{2}t}{sin\frac{t}{2}}dt$ (đpcm)




#559348 Tính tổng của chuỗi bằng phương trình vi phân

Đã gửi bởi sinh vien on 14-05-2015 - 18:18 trong Giải tích

Bài toán (Putnam 1950) Chứng minh rằng:

         $x+\frac{x^{3}}{1.3}+\frac{x^{5}}{1.3.5}+\frac{x^{7}}{1.3.5.7}+...+\frac{x^{2k+1}}{1.3.5...(2k+1)}+...=e^{\frac{x^{2}}{2}}\int_{0}^{x}e^{-\frac{t^{2}}{2}}dt$




#562123 Tính số chiều của các không gian các vector con

Đã gửi bởi sinh vien on 28-05-2015 - 16:45 trong Đại số tuyến tính, Hình học giải tích

Bài toán.(ĐH - Toronto 2015 ) Tính số chiều của không gian vector con sinh bởi $\left \{ (\sigma (1);\sigma (2);...;\sigma (n)) \right \}$. trong đó $\sigma$  là một song ánh từ $\left \{ 1,2...n \right \}\rightarrow \left \{ 1,2,...,n \right \}$.

   Đáp số : n




#560962 Bài toán tô màu - hệ phương trình

Đã gửi bởi sinh vien on 22-05-2015 - 19:23 trong Thảo luận về các kì thi, các kì kiểm tra Toán sinh viên

Bài toán (AVJ-2009) Cho k và n là hai số nguyên dương thỏa $k\leq n-1$ . Đặt $S=\left \{ 1,2,...,n \right \}$ và $A_{1},A_{2},..A_{k}$ là các tập con khác rỗng của S. Chứng minh rằng ta có thể tô màu một số phần tử của S bằng một trong hai màu xanh hoặc đỏ ( có thể có những phần tử không được tô màu) sao cho

 i) Một phần tử bất kỳ của S hoặc không được tô màu hoặc được tô màu đỏ hoặc được tô màu xanh

 ii) Có ít nhất một phần tử của S được tô màu

 iii) Mọi tập $A_{i} (i=1,2...k)$ hoặc chứa toàn các số không được tô màu hoặc chứa ít nhất một cặp số mà mỗi số được tô bởi hai màu khác nhau




#560220 Sử dụng tích phân chứng minh bất đẳng thức

Đã gửi bởi sinh vien on 18-05-2015 - 20:18 trong Giải tích

Bài toán(Hilbert).Chứng minh :

  Với mọi số thực $a_{1},a_{2},...,a_{n}$ ta luôn có

                   $\sum_{i=1}^{n}\sum_{j=1}^{n}\frac{a_{i}a_{j}}{i+j}\leq \pi \sum_{i=1}^{n}a_{i}^{2}$

Bài toán ( Frilz Carlson) Với mọi số thực $a_{1},a_{2},...,a_{n}$ ta luôn có

                          $\pi ^{2}(a_{1}^{2}+a_{2}^{2}+...+a_{n}^{2})(a_{1}^{2}+4a_{2}^{2}+...+n^{2}a_{n}^{2})\leq (a_{1}+a_{2}+...+a_{n})^{4}$

Bài toán.  Chứng minh rằng với a,b,c,x,y,z,t là các số thực dương sao cho $1\leq x,y,z\leq 4$ ta luôn có

       $\frac{x}{(2a)^{t}}+\frac{y}{(2b)^{t}}+\frac{z}{(2c)^{t}}\geq \frac{y+z-x}{(b+c)^{t}}+\frac{z+x-y}{(c+a)^{t}}+\frac{x+y-z}{(a+b)^{t}}$

   Nguồn các bài toán : 2 bài đầu mình trích ra từ cuốn '' Problem from Book '' của tác giả Titu Andresscu.

Bài toán còn lại là từ một cuộc thi tại cuộc thi Annual Vojtech Jarnik




#688167 Đường đi ngẫu nhiên trên đồ thị

Đã gửi bởi sinh vien on 20-07-2017 - 17:28 trong Tài liệu, chuyên đề Toán cao cấp

File gửi kèm  Đường đi ngẫu nhiên trên đồ thị.pdf   292.76K   100 Số lần tải




#557834 CMR tồn tại hằng số B dương thỏa $\sum_{i,j=1}^{n...

Đã gửi bởi sinh vien on 04-05-2015 - 18:56 trong Giải tích

Bài toán. (PUTNAM 2011) Cho $a_{1},a_{2},...a_{n}$ là các số thực thỏa mãn 

          $\int_{-\infty }^{\infty }\left ( \sum_{i=1}^{n}\frac{1}{1+\left ( x-a_{i} \right )^{2}}\right )^{2}dx\leqslant An$ với mọi n , trong đó A là một hằng số dương .

 Chứng minh rằng tồn tại hằng số B dương thỏa $\sum_{i,j=1}^{n}\left ( 1+\left ( a_{i}-a_{j} \right )^{2} \right )\geq Bn^{3}$.

Lời giải. 

 Với $a\neq 0$, $\int_{-\infty }^{\infty }\frac{du}{\left ( 1+\left ( u+a \right )^{2} \right )\left ( 1+\left ( u-a \right )^{2} \right )}=\frac{1}{4a(1+a^{2})}\int_{-\infty }^{\infty }\left [ \frac{u+a}{1+\left ( u+a\right )^{2}}-\frac{u-a}{1+\left ( u-a \right )^{2}} \right ]du$$=\frac{1}{4a\left ( 1+a^{2} \right )}\left [ \frac{1}{2}ln\frac{1+\left ( u+a \right )^{2}}{1+(u-a)^{2}} +aarctan(u+a)+aarctan(u-a)\right ]_{-\infty }^{\infty }$$=\frac{\pi }{2(1+a^{2})}$.

   Sử dụng phép đổi biến: $x=u+\frac{a+b}{2}$ ta được 

$\int_{-\infty }^{\infty }\frac{dx}{(1+(x-a)^{2})(1+(x-b)^{2})}=\frac{2\pi }{4+(a-b)^{2}}$, trong đó $a\neq b$

Trong trường hợp a=b 

  $\int_{-\infty }^{\infty }\frac{dx}{(1+(x-a)^{2})^{2}}=\int_{-\frac{\pi }{2}}^{\frac{\pi }{2}}\frac{\frac{1}{cos^{2}\theta }d\theta }{\frac{1}{cos^{4}\theta }}=\int_{-\frac{\pi }{2}}^{\frac{\pi }{2}}cos^{2}\theta d\theta =\frac{\pi }{2}$

 nên côn thức trên cũng đúng với mọi cặp số a,b.

 Ta có

$An\geq \int_{-\infty }^{\infty }\left [ \sum_{i=1}^{n}\frac{1}{1+(x-a_{i})^{2}}\right ]^{2}dx=\sum_{i,j=1}^{n}\int_{-\infty }^{\infty }\frac{dx}{(1+(x-a_{i})^{2})(1+(x-a_{j})^{2})}$$=\sum_{i,j=1}^{n}\frac{2\pi }{4+(a_{i}-a_{j})^{2}}\geq \frac{\pi }{2}\sum_{i,j=1}^{n}\frac{1}{1+\left ( a_{i}-a_{j} \right )^{2}}$

 Sử dụng bất đẳng thức quen thuộc Cauchy- Schwarz 

$n^{4}\leq \left [ \sum_{i,j=1}^{n}\frac{1}{1+(a_{i}-a_{j})^{2}} \right ]\sum_{i,j=1}^{n}(1+(a_{i}-a_{j})^{2})\leq \frac{2An}{\pi }\sum_{i,j=1}^{n}(1+(a_{i}-a_{j})^{2})$

  Chọn $B=\frac{\pi }{2A}$ . thì điều kiện bài toán được thỏa mãn




#559330 Một minh họa thú vị

Đã gửi bởi sinh vien on 14-05-2015 - 17:16 trong Giải tích

Bài toán.(Putnam 1958) Chứng minh rằng phương trình tích phân sau nếu  có nghiệm  thuộc lớp hàm liên tục trên hình vuông $[0,1]\times [0,1]$.

                                $f(x,y)=1+\int_{0}^{x}\int_{0}^{y}f(u,v)dudv$

thì nghiệm đó là duy nhất.

Lời giải . Gỉa sử $f_{1}(x,y),f_{2}(x,y)$ là hai nghiệm bất kỳ của phương trình tích phân đã cho. Đặt $g(x,y)=f_{1}(x,y)-f_{2}(x,y)$. Ta thấy

$g(x,y)=\left ( 1+\int_{0}^{x}\int_{0}^{y}f_{1}(u,v)dudv \right )-\left ( 1+\int_{0}^{x}\int_{0}^{y}f_{2}(u,v)dudv \right )$

          $=\int_{0}^{x}\int_{0}^{y}\left ( f_{1}(u,v)-f_{2}(u,v) \right )dudv=\int_{0}^{x}\int_{0}^{y}g(x,y)dxdy$

Dễ dàng nhận thấy g(x,y) cũng là một hàm liên tục trên $[0,1]\times [0,1]$ nên g(x,y) bị chặn

hay nói cách khác tồn tại M sao cho $\left | g(x,y) \right |\leq M$ với mọi $(x,y)\in [0,1]\times [0,1]$

 Ta sẽ chứng minh quy nạp kết quả then chốt sau : $\left | g(x,y) \right |\leq M\frac{x^{n}}{n!}\frac{y^{n}}{n!}$ với mọi số nguyên dương n

 Với n=1 , kết quả hiển nhiên.

Gỉa sử kết quả đúng với n=k . Ta cần chứng minh nó cũng đúng cho n=k+1. Ta thấy

$\left | g(x,y) \right |\leqslant \left | \int_{0}^{x}\int_{0}^{y}g(u,v)dudv \right |\leq \int_{0}^{x}\int_{0}^{y}M\frac{u^{k}}{k!}\frac{v^{k}}{k!}dudv=M\frac{x^{k+1}}{(k+1)!}\frac{y^{k+1}}{(k+1)!}$

 Vậy kết quả cũng đúng với n=k+1

  Từ kết quả trên nếu ta cố định bộ số (x,y) , qua giới hạn khi $n\rightarrow \infty$ , ta suy ra $\left | g(x,y) \right |\leq 0\Rightarrow g(x,y)=0$

    Nhận xét này chứng tỏ rằng phương trình tích phân đã cho không thể có hai nghiệm phân biệt .




#719593 bài toán năm vật thể đều

Đã gửi bởi sinh vien on 19-01-2019 - 22:20 trong Tài liệu, chuyên đề Toán cao cấp

File gửi kèm  Bài toán năm vật thể đều.pdf   342.4K   77 Số lần tải




#560140 Bài toán tô màu các số

Đã gửi bởi sinh vien on 18-05-2015 - 12:24 trong Thảo luận về các kì thi, các kì kiểm tra Toán sinh viên

Bài toán. Ta tô màu các số thuộc tập hợp {1,2,...,n} bằng một trong 6 màu khác nhau.

 Đặt

       $S=\left \{ (x,y,z)\in \left \{ 1,2...n \right \}^{3}:x+y+z\equiv 0\right \}$ và x,y,z có cùng màu

        $D=\left \{ (x,y,z)\in \left \{ 1,2,...n \right \}^{3} :x+y+z\equiv 0\right \}$ và x,y,z có màu đôi một khác nhau

  Chứng minh rằng : $\left | D \right |\leq 2\left | S \right |+\frac{n^{2}}{2}$

 Nguồn : Từ cuộc thi Annual Vojtech Jarnik




#559232 Các bài toán về hàm hình học

Đã gửi bởi sinh vien on 13-05-2015 - 21:07 trong Hình học

Bài toán (Putnam 2009). Gọi $\mathbb{P}$ là tập hợp các điểm trên mặt phẳng , $f:\mathbb{P}\rightarrow \mathbb{R}$ là một hàm số nhận giá trị thực thỏa mãn $f(A)+f(B)+f(C)+f(D)=0$, với mọi hình vuông ABCD .

    Chứng minh rằng f(P)=0 với mọi P

Lời giải. Gọi P là một điểm bất kỳ thuộc $\mathbb{P}$. Gọi ABCD là hình vuông nhận P làm tâm . Gọi E,F,G,H lần lượt là trung điểm của các cạnh AB ,BC,CD,DA. 

Từ điều kiện đề bài ta có 

   0=f(A)+f(B)+f(C)+f(D)

   0=f(E)+f(F)+f(G)+f(H)

   0=f(A)+f(E)+f(P)+f(H)

   0=f(B)+f(F)+f(P)+f(E)

   0=f(C)+f(G)+f(P)+f(F)

   0=f(D)+f(H)+f(P)+f(G)

 Cộng bồn đẳng thức cuối kết hợp với hai đẳng thức đầu ta có ngay kết quả